Làm thế nào chúng ta có thể ràng buộc xác suất rằng một biến ngẫu nhiên là tối đa?


21

Giả sử chúng ta có biến ngẫu nhiên độc lập X_1 , \ ldots , X_n với phương tiện hữu hạn \ mu_1 \ leq \ ldots \ leq \ mu_N và phương sai \ sigma_1 ^ 2 , \ ldots , \ sigma_N ^ 2 . Tôi đang tìm kiếm giới hạn không phân phối với xác suất rằng bất kỳ X_i \ neq X_N nào lớn hơn tất cả các X_j khác , j \ neq i .X 1 LọNX1Xnσ 2 1 ... σ 2 N X iX N X jμ1μNσ12σN2XiXNXjji

Nói cách khác, nếu để đơn giản, chúng tôi giả sử các phân phối của Xi là liên tục (sao cho P(Xi=Xj)=0 ), tôi đang tìm kiếm giới hạn trên:

P(Xi=maxjXj).
Nếu N=2 , chúng ta có thể sử dụng bất đẳng thức của Ch Quashev để có được: \ P (X_1 = \ max_j X_j) = \ P (X_1> X_2) \ leq \ frac {\ sigma_1 ^ 2 + \ sigma_2 ^ 2} {\ sigma_1 ^ 2 + \ sigma_2 ^ 2 + (\ mu_1 - \ mu_2) ^ 2} \ không gian.
P(X1= =tối đajXj)= =P(X1>X2)σ12+σ22σ12+σ22+(μ1-μ2)2.
Tôi muốn tìm một số giới hạn đơn giản (không nhất thiết phải chặt chẽ) cho N chung N, nhưng tôi không thể tìm thấy kết quả (về mặt thẩm mỹ) cho N chung N.

Xin lưu ý rằng các biến không được coi là iid. Bất kỳ đề xuất hoặc tài liệu tham khảo cho công việc liên quan đều được chào đón.


Cập nhật: nhắc lại rằng theo giả định, μjμtôi . Sau đó, chúng tôi có thể sử dụng ràng buộc ở trên để đến:

P(Xtôi= =tối đajXj)tối thiểuj>tôiσtôi2+σj2σtôi2+σj2+(μj-μtôi)2σtôi2+σN2σtôi2+σN2+(μN-μtôi)2.
Điều này ngụ ý:
(μN-μtôi)P(Xtôi= =tối đajXj)(μN-μtôi)σtôi2+σN2σtôi2+σN2+(μN-μtôi)212σtôi2+σN2.
Đến lượt nó, ngụ ý:
i=1NμiP(Xi=maxjXj)μNN2i=1N1(σi2+σN2).
Tôi bây giờ tự hỏi liệu điều này ràng buộc có thể được cải thiện một cái gì đó mà không phụ thuộc tuyến tính trên N . Chẳng hạn, việc giữ như sau:
i=1NμiP(Xi=maxjXj)μNi=1Nσi2?
Và nếu không, những gì có thể là một ví dụ?

3
Ràng buộc này có thể chặt chẽ hơn nếu bạn sử dụng các chỉ số cung cấp cho bạn nhỏ trên ràng buộc thay vì . Lưu ý rằng giá trị này phụ thuộc vào cả giá trị trung bình và phương sai. NjN

5
@MichaelCécick: Tôi không tin đó là chính xác. Ví dụ, giả sử chúng ta có ba bản phân phối thống nhất trên . Sau đó, nếu tôi không nhầm, , trong khi . Tôi không biết bạn có ý định viết , nhưng ví dụ tương tự cho thấy nó vẫn không hợp lệ. P ( X 1 < max j X j ) = 2 / 3 P ( X 1 < X 2 ) = P ( X 1 < X 3 ) = 1 / 2 P ( X i > max j X j )[0,1]P(X1<maxjXj)=2/3P(X1<X2)=P(X1<X3)=1/2P(Xi>maxjXj)
MLS

2
@Michael: Điều đó vẫn không đúng, thật không may. Các sự kiện cho cố định không độc lập. iAj={Xi>Xj} i
Đức hồng y

2
@cardinal: Trong số những thứ khác, nó liên quan đến những tên cướp đa vũ trang. Nếu bạn chọn một nhánh dựa trên phần thưởng trước đó, thì xác suất bạn chọn được cánh tay tốt nhất là bao nhiêu (đó sẽ là trong ký hiệu ở trên) và chúng ta có thể ràng buộc tổn thất dự kiến ​​khi chọn phụ cánh tay tối ưu? P(XN=maxjXj)
MLS

2
Crossposted to MathOverflow: mathoverflow.net/questions/99313
hồng y

Câu trả lời:


1

Bạn có thể sử dụng bất đẳng thức đa biến của Ch Quashev.

Trường hợp hai biến

Đối với một tình huống duy nhất, so với , tôi đến tình huống tương tự như nhận xét của Jochen vào ngày 4 tháng 11 năm 2016X 2X1X2

1) Nếu thì P ( X 1 > X 2 ) ( σ 2 1 + σ 2 2 ) / ( μ 1 - μ 2 ) 2μ1<μ2P(X1>X2)(σ12+σ22)/(μ1μ2)2

(và tôi cũng tự hỏi về sự phát sinh của bạn)

Đạo hàm của phương trình 1

  • sử dụng biến mớiX1X2
  • biến đổi nó sao cho nó có giá trị trung bình bằng không
  • lấy giá trị tuyệt đối
  • áp dụng bất đẳng thức của Ch Quashev

P(X1>X2)=P(X1X2>0)=P(X1X2(μ1μ2)>(μ1μ2))P(|X1X2(μ1μ2)|>μ2μ1)σ(X1-X2-(μ1-μ2))2(μ2-μ1)2= =σX12+σX22(μ2-μ1)2

Trường hợp đa biến

Bất đẳng thức trong phương trình (1) có thể được thay đổi thành trường hợp đa biến bằng cách áp dụng nó cho nhiều biến được chuyển đổi cho mỗi (lưu ý rằng đây là những tương quan).i < n(Xn-Xtôi)tôi<n

Một giải pháp cho vấn đề này (đa biến và tương quan) đã được mô tả bởi I. Olkin và JW Pratt. 'Một bất đẳng thức đa biến Tchithercheff' trong Biên niên sử thống kê toán học, tập 29 trang 226-234 http://projecteuclid.org/euclid.aoms/1177706720

Lưu ý định lý 2.3

P(|yi|kiσi for some i)=P(|xi|1 for some i)(u+(ptu)(p1))2p2

trong đó số lượng biến, và .pt=ki2u=ρij/(kikj)

Định lý 3.6 cung cấp một ràng buộc chặt chẽ hơn, nhưng ít dễ tính toán hơn.

Chỉnh sửa

Một ràng buộc sắc nét hơn có thể được tìm thấy bằng cách sử dụng bất đẳng thức của Cantelli . Bất bình đẳng đó là kiểu mà bạn đã dùng trước và cung cấp cho bạn những ranh giới đó là sắc nét hơn .(σ12+σ22)/(σ12+σ22+(μ1μ2)2)(σ12+σ22)/(μ1μ2)2

Tôi đã không dành thời gian để nghiên cứu toàn bộ bài viết, nhưng dù sao, bạn có thể tìm thấy một giải pháp ở đây:

AW Marshall và I. Olkin 'Một sự bất bình đẳng một chiều của loại Ch Quashev ' trong Biên niên sử Thống kê toán học tập 31 trang 488-491 https://projecteuclid.org/euclid.aoms/1177705913

(lưu ý sau: Bất đẳng thức này là cho các mối tương quan bằng nhau và không đủ trợ giúp. Nhưng dù sao, vấn đề của bạn, để tìm ra ràng buộc sắc nét nhất, bằng với bất đẳng thức Cantelli đa biến, tổng quát hơn. Tôi sẽ ngạc nhiên nếu giải pháp không tồn tại)


Bạn có thể cung cấp một tuyên bố rõ ràng về bất bình đẳng đa biến Ch Quashev?
whuber

1
Tôi đã chỉnh sửa giải pháp cung cấp toàn bộ định lý.
Sextus Empiricus

-1

Tôi đã tìm thấy một định lý có thể giúp bạn và sẽ cố gắng điều chỉnh nó theo nhu cầu của bạn. Giả sử bạn có:

exp(tE(mmộtx1tôinXtôi))

Sau đó, bởi bất đẳng thức của Jensen (vì exp (.) Là một hàm lồi), chúng ta nhận được:

exp(tE(mmộtx1tôinXtôi))E(exp(tmmộtx1tôinXtôi))= =E(mmộtx1tôin exp(tXtôi))Σtôi= =1nE(exp(tXtôi)

Bây giờ đối với bạn phải cắm bất cứ chức năng tạo khoảnh khắc nào của biến ngẫu nhiên là (vì đó chỉ là định nghĩa của mgf). Sau đó, sau khi làm như vậy (và có khả năng đơn giản hóa thuật ngữ của bạn), bạn lấy thuật ngữ này và lấy nhật ký và chia cho nó để bạn nhận được một tuyên bố về thuật ngữ . Sau đó, bạn có thể chọn t với một số giá trị tùy ý (tốt nhất sao cho số hạng nhỏ để giới hạn chặt chẽ).exp(tXtôiXtôiE(mmộtx1tôinXtôi)

Sau đó, bạn có một tuyên bố về giá trị dự kiến ​​của tối đa trên n rvs. Để có được tuyên bố về xác suất rằng mức tối đa của các rv đó lệch khỏi giá trị mong đợi này, bạn chỉ cần sử dụng bất đẳng thức của Markov (giả sử rằng rv của bạn không âm) hoặc rv khác, cụ thể hơn, áp dụng cho rv cụ thể của bạn.

Khi sử dụng trang web của chúng tôi, bạn xác nhận rằng bạn đã đọc và hiểu Chính sách cookieChính sách bảo mật của chúng tôi.
Licensed under cc by-sa 3.0 with attribution required.